K
Khách

Hãy nhập câu hỏi của bạn vào đây, nếu là tài khoản VIP, bạn sẽ được ưu tiên trả lời.

NV
27 tháng 9 2019

\(\frac{1}{ab}+\frac{1}{bc}+\frac{1}{ca}=6\)

Đặt \(\left(\frac{1}{a};\frac{1}{b};\frac{1}{c}\right)=\left(x;y;z\right)\Rightarrow xy+yz+zx=6\)

\(P=\frac{x^3}{y+2z}+\frac{y^3}{z+2x}+\frac{z^3}{x+2y}=\frac{x^4}{xy+2xz}+\frac{y^4}{yz+2xy}+\frac{z^4}{zx+2yz}\)

\(P\ge\frac{\left(x^2+y^2+z^2\right)^2}{3\left(xy+yz+zx\right)}\ge\frac{xy+yz+zx}{3}=2\)

Dấu "=" xảy ra khi \(x=y=z=\sqrt{2}\)

NV
5 tháng 11 2019

\(a+b+c=6abc\Leftrightarrow\frac{1}{ab}+\frac{1}{bc}+\frac{1}{ca}=6\)

Đặt \(\left(\frac{1}{a};\frac{1}{b};\frac{1}{c}\right)=\left(x;y;z\right)\Rightarrow xy+yz+zx=6\)

\(P=\frac{x^3}{y+2z}+\frac{y^3}{z+2x}+\frac{z^3}{x+2y}=\frac{x^4}{xy+2zx}+\frac{y^4}{yz+2xy}+\frac{z^4}{zx+2yz}\)

\(P\ge\frac{\left(x^2+y^2+z^2\right)^2}{3\left(xy+yz+zx\right)}\ge\frac{\left(xy+yz+zx\right)^2}{3\left(xy+yz+zx\right)}=\frac{xy+yz+zx}{3}=2\)

Dấu "=" xảy ra khi \(x=y=z=\sqrt{2}\) hay \(a=b=c=\frac{1}{\sqrt{2}}\)

NM
8 tháng 5 2021

Đặt \(x=\frac{1}{a}, y=\frac{1}{b}, z=\frac{1}{c}, \Rightarrow x+y+z=2\)

Suy ra    \(\frac{1}{a\left(2a-1\right)^2}+\frac{1}{b\left(2b-1\right)^2}+\frac{1}{c\left(2c-1\right)^2}=\frac{x^3}{\left(2-x\right)^2}+\frac{y^3}{\left(2-y\right)^2}+\frac{z^3}{\left(2-z\right)^2}\)

Ta có \(\frac{x^3}{\left(2-x\right)^2}+\frac{2-x}{8}+\frac{2-x}{8}\ge3\sqrt[3]{\frac{x^3}{\left(2-x\right)^2} .\frac{2-x}{8}.\frac{2-x}{8}}=\frac{3x}{4}.\)

\(\Rightarrow\frac{x^3}{\left(2-x\right)^2}\ge x-\frac{1}{2}\)\(\Rightarrow\frac{x^3}{\left(2-x\right)^2}+\frac{y^3}{\left(2-y\right)^2}+\frac{z^3}{\left(2-z\right)^2}\ge x+y+z-\frac{3}{2}=2-\frac{3}{2}=\frac{1}{2}\)

dấu "=" xảy ra khi \(x=y=z=\frac{2}{3}\)hay \(a=b=c=\frac{3}{2}\)

9 tháng 6 2020

Không mất tính tổng quát, chuẩn hóa a + b + c = 1

Khi đó, ta cần chứng minh: \(\frac{\left(a+1\right)^2}{2a^2+\left(1-a\right)^2}+\frac{\left(b+1\right)^2}{2b^2+\left(1-b\right)^2}+\frac{\left(c+1\right)^2}{2c^2+\left(1-c\right)^2}\le8\)

Xét bất đẳng thức phụ: \(\frac{\left(x+1\right)^2}{2x^2+\left(1-x\right)^2}\le4x+\frac{4}{3}\)(*)

Thật vậy: (*)\(\Leftrightarrow\frac{\left(3x-1\right)^2\left(4x+1\right)}{2x^2+\left(1-x\right)^2}\ge0\)*đúng*

Áp dụng, ta được: \(\frac{\left(a+1\right)^2}{2a^2+\left(1-a\right)^2}+\frac{\left(b+1\right)^2}{2b^2+\left(1-b\right)^2}+\frac{\left(c+1\right)^2}{2c^2+\left(1-c\right)^2}\)\(\le4\left(a+b+c\right)+4=4.1+4=8\)

Vậy bất đẳng thức được chứng minh

Đẳng thức xảy ra khi a = b = c

25 tháng 7 2019

Chuẩn hóa ta có : \(a+b+c=3\)

=> \(\frac{\left(2a+b+c\right)^2}{2a^2+\left(b+c\right)^2}=\frac{\left(a+3\right)^2}{2a^2+\left(3-a\right)^2}=\frac{a^2+6a+9}{3\left(a^2-2a+3\right)}\)

Xét\(\frac{a^2+6a+9}{3\left(a^2-2a+3\right)}\le\frac{4}{3}a+\frac{4}{3}\)

<=> \(a^2+6a+9\le4\left(a+1\right)\left(a^2-2a+3\right)\)

<=> \(4a^3-5a^2-2a+3\ge0\)

<=> \(\left(a-1\right)^2\left(4a+3\right)\ge0\)luôn đúng

Khi đó 

\(VT\le\frac{4}{3}\left(a+b+c\right)+4=\frac{4}{3}.3+4=8\)(ĐPCM)

Dấu bằng xảy ra khi a=b=c

4 tháng 4 2020

Hôm qua em không có online. Bài này căng não@@

Đặt \(p=a+b+c;q=ab+bc+ca;r=abc\Rightarrow q=3\) thì \(p^2\ge3q=9\Rightarrow p\ge3\)

Chú ý: \(-4p^3r + p^2q^2 + 18pqr - 4q^3 - 27r^2=(a-b)^2 (b-c)^2 (c-a)^2 \geq 0\)

\(\Rightarrow\) \(1/27(-2p^3-2\sqrt{(p^2-3q)^3}+9pq) \leq r \leq 1/27(-2p^3+2\sqrt{(p^2-3q)^3}+9pq)\)

Hay là: \(\frac{1}{27}\left(-2p^3-2\sqrt{\left(p^2-9\right)^3}+27p\right)\le r\le\frac{1}{27}\left(-2p^3+2\sqrt{\left(p^2-9\right)^3}+27p\right)\)

Nếu \(a\ge b\ge c\Rightarrow a^2b+b^2c+c^2a\ge ab^2+bc^2+ca^2\)

\(\Rightarrow a^2b+b^2c+c^2a\ge\frac{1}{2}\Sigma ab\left(a+b\right)=\frac{1}{2}\left(pq-3r\right)=\frac{3}{2}\left(p-3r\right)\)

Do đó: \(P\ge\frac{1}{2}\left(p-3r\right)+\sqrt[3]{9p}\ge\frac{1}{2}\left(p-\frac{1}{27}\left(-2p^3+2\sqrt{\left(p^2-9\right)^3}+27p\right)\right)+3\)

\(\ge\frac{1}{27}p^3-\frac{1}{27}\sqrt{\left(p^2-9\right)^3}+3=f\left(p\right)\). Dễ thấy khi p tăng thì f(p) tăng.

Do đó f(p) đạt giá trị nhỏ nhất khi p đạt giá trị nhỏ nhất. Hay là: \(f\left(p\right)\ge f\left(3\right)=4=VP\)

Trường hợp còn lại tối về em đăng, đang bận!

4 tháng 4 2020

Nếu \(a\le b\le c\Rightarrow\left(a-b\right)\left(b-c\right)\left(a-c\right)\le0\)

\(\Rightarrow\left(a-b\right)\left(b-c\right)\left(a-c\right)=-\left|\left(a-b\right)\left(b-c\right)\left(a-c\right)\right|=-\sqrt{\left(a-b\right)^2\left(b-c\right)^2\left(c-a\right)^2}\)

\(=-\sqrt{-4p^3r + p^2q^2 + 18pqr - 4q^3 - 27r^2}\)

---------------------------------------------------------------------------------------------------------

Chú ý: \(-4p^3r + p^2q^2 + 18pqr - 4q^3 - 27r^2=(a-b)^2 (b-c)^2 (c-a)^2 \geq 0\)

\(\Rightarrow\) \(1/27(-2p^3-2\sqrt{(p^2-3q)^3}+9pq) \leq r \leq 1/27(-2p^3+2\sqrt{(p^2-3q)^3}+9pq)\)

Hay là: \(\frac{1}{27}\left(-2p^3-2\sqrt{\left(p^2-9\right)^3}+27p\right)\le r\le\frac{1}{27}\left(-2p^3+2\sqrt{\left(p^2-9\right)^3}+27p\right)\)

Ta có: \(2\left(a^2b+b^2c+c^2a\right)=\Sigma ab\left(a+b\right)+\left(a-b\right)\left(b-c\right)\left(a-c\right)\)

\(=pq-3r-\sqrt{-4p^3r + p^2q^2 + 18pqr - 4q^3 - 27r^2}\)

\(=3p-3r-\sqrt{-4p^3r + 9p^2 + 54pr - 108 - 27r^2}\)

Do đó: \(a^2b+b^2c+c^2a\)\(=\frac{3p-3r-\sqrt{-4p^3r + 9p^2 + 54pr - 108 - 27r^2}}{2}\)

Do đó: \(P\)\(=\frac{3p-3r-\sqrt{-4p^3r + 9p^2 + 54pr - 108 - 27r^2}}{6}\)\(+\sqrt[3]{9p}\ge4\)

\(\Leftrightarrow\frac{3p-3r}{6}+\sqrt[3]{9p}\ge4+\)\(\frac{\sqrt{-4p^3r + 9p^2 + 54pr - 108 - 27r^2}}{6}\)

Or \(3p-3r+6\sqrt[3]{9p}-24\ge\)\(\sqrt{-4p^3r + 9p^2 + 54pr - 108 - 27r^2}\)

Vì: \(VT=3p-3r+6\sqrt[3]{9p}-24\ge3p-\frac{pq}{3}+18-24=0\)

Nên bất đẳng thức trên tương đương:

\(\left(3p-3r+6\sqrt[3]{9p}-24\right)^2\ge\) \(-4p^3r + 9p^2 + 54pr - 108 - 27r^2\)

Em chịu thua :( @Akai Haruma @Nguyễn Việt Lâm giúp em với ạ.